LEIK FNP PRACTICE QUESTIONS WITH ANSWERS AND RATIONALES 2024 REVISED AND GRADED A+. | Exams Nursing | Docsity (2024)

  • Prepare for your exams

  • Get points

  • Guidelines and tips

  • Sell on Docsity
Log inSign up

LEIK FNP PRACTICE QUESTIONS WITH ANSWERS AND RATIONALES 2024 REVISED AND GRADED A+. | Exams Nursing | Docsity (2)

LEIK FNP PRACTICE QUESTIONS WITH ANSWERS AND RATIONALES 2024 REVISED AND GRADED A+. | Exams Nursing | Docsity (3)

Prepare for your exams

Study with the several resources on Docsity

LEIK FNP PRACTICE QUESTIONS WITH ANSWERS AND RATIONALES 2024 REVISED AND GRADED A+. | Exams Nursing | Docsity (4)

Earn points to download

Earn points by helping other students or get them with a premium plan

LEIK FNP PRACTICE QUESTIONS WITH ANSWERS AND RATIONALES 2024 REVISED AND GRADED A+. | Exams Nursing | Docsity (5)

Guidelines and tips

Sell on Docsity
Log inSign up

LEIK FNP PRACTICE QUESTIONS WITH ANSWERS AND RATIONALES 2024 REVISED AND GRADED A+. | Exams Nursing | Docsity (7)

Prepare for your exams

Study with the several resources on Docsity

Find documentsPrepare for your exams with the study notes shared by other students like you on DocsitySearch Store documentsThe best documents sold by students who completed their studies

Search through all study resources

Docsity AINEWSummarize your documents, ask them questions, convert them into quizzes and concept mapsExplore questionsClear up your doubts by reading the answers to questions asked by your fellow students

LEIK FNP PRACTICE QUESTIONS WITH ANSWERS AND RATIONALES 2024 REVISED AND GRADED A+. | Exams Nursing | Docsity (8)

Earn points to download

Earn points by helping other students or get them with a premium plan

Share documents20 PointsFor each uploaded documentAnswer questions5 PointsFor each given answer (max 1 per day)

All the ways to get free points

Get points immediatelyChoose a premium plan with all the points you need

Study Opportunities

Search for study opportunitiesNEWConnect with the world's best universities and choose your course of study

Community

Ask the communityAsk the community for help and clear up your study doubts University RankingsDiscover the best universities in your country according to Docsity users

Free resources

Our save-the-student-ebooks!Download our free guides on studying techniques, anxiety management strategies, and thesis advice from Docsity tutors

From our blog

Exams and Study

Go to the blog

The University of Edinburgh (Edin)Nursing

test the knowledge and clinical reasoning skills necessary for Family Nurse Practitioners (FNPs). Questions cover a wide range of topics including health assessment, diagnosis, treatment plans, pharmacology, and patient education. Each answer includes a rationale to reinforce understanding of clinical decision-making and evidence-based practice, ensuring FNPs are well-prepared for real-world healthcare scenarios and achieving a high level of proficiency in their field.

Typology: Exams

2023/2024

1 / 240

LEIK FNP PRACTICE QUESTIONS WITH ANSWERS AND RATIONALES 2024 REVISED AND GRADED A+. | Exams Nursing | Docsity (12)

Related documents

LEIK FNP PRACTICE QUESTIONS WITH ANSWERS AND RATIONALES 2024 REVISED AND GRADED A+. | Exams Nursing | Docsity (13)

NAVLE EXAM PRACTICE EXAM 2024 | 200+ QUESTIONS AND CORRECT ANSWERS WITH RATIONALES

LEIK FNP PRACTICE QUESTIONS WITH ANSWERS AND RATIONALES 2024 REVISED AND GRADED A+. | Exams Nursing | Docsity (14)

LEIK FAMILY NURSING PRACTITIONER PRACTICE QUESTIONS WITH ANSWERS AND RATIONALES LATEST UPD

LEIK FNP PRACTICE QUESTIONS WITH ANSWERS AND RATIONALES 2024 REVISED AND GRADED A+. | Exams Nursing | Docsity (15)

MED SURG GASTROINTESTINAL NCLEX QUESTIONS WITH ANSWERS AND RATIONALES 2024 REVISED GRADED

LEIK FNP PRACTICE QUESTIONS WITH ANSWERS AND RATIONALES 2024 REVISED AND GRADED A+. | Exams Nursing | Docsity (16)

LEIK Family Nursing Practitioner Practice Questions with Answers and Rationales 2024 BEST

LEIK FNP PRACTICE QUESTIONS WITH ANSWERS AND RATIONALES 2024 REVISED AND GRADED A+. | Exams Nursing | Docsity (17)

LEIK FNP Practice Questions with Answers and Rationales 2024 FINAL EXAM UPDATE

LEIK FNP PRACTICE QUESTIONS WITH ANSWERS AND RATIONALES 2024 REVISED AND GRADED A+. | Exams Nursing | Docsity (18)

HEALTH ASSESSMENT NCLEX QUESTIONS WITH ANSWERS AND RATIONALES 2024 REVISED GRADED A+ ACTUA

LEIK FNP PRACTICE QUESTIONS WITH ANSWERS AND RATIONALES 2024 REVISED AND GRADED A+. | Exams Nursing | Docsity (19)

ILTS Elementary Education (Grades 1-6) Questions With Answers And Rationales 2024 Revised

LEIK FNP PRACTICE QUESTIONS WITH ANSWERS AND RATIONALES 2024 REVISED AND GRADED A+. | Exams Nursing | Docsity (20)

LEIK Family Nursing Practitioner Practice Questions with Answers and Rationales.

LEIK FNP PRACTICE QUESTIONS WITH ANSWERS AND RATIONALES 2024 REVISED AND GRADED A+. | Exams Nursing | Docsity (21)

LEIK Family Nursing Practitioner Practice Questions with Answers and Rationales.

LEIK FNP PRACTICE QUESTIONS WITH ANSWERS AND RATIONALES 2024 REVISED AND GRADED A+. | Exams Nursing | Docsity (22)

LEIK FNP PRACTICE QUESTIONS WITH ANSWERS AND RATIONALES 2024 REVISED AND GRADED A+. | Exams Nursing | Docsity (23)

LEIK FNP (642 Practice Questions with 100% Verified Answers and Rationales).

LEIK FNP PRACTICE QUESTIONS WITH ANSWERS AND RATIONALES 2024 REVISED AND GRADED A+. | Exams Nursing | Docsity (24)

LEIK FNP (642 Practice Questions with 100% Verified Answers and Rationales).

LEIK FNP PRACTICE QUESTIONS WITH ANSWERS AND RATIONALES 2024 REVISED AND GRADED A+. | Exams Nursing | Docsity (25)

LEIK FNP (642 Practice Questions with 100% Verified Answers and Rationales).

LEIK FNP PRACTICE QUESTIONS WITH ANSWERS AND RATIONALES 2024 REVISED AND GRADED A+. | Exams Nursing | Docsity (26)

LEIK FNP (642 Practice Questions with 100% Verified Answers and Rationales).

LEIK FNP PRACTICE QUESTIONS WITH ANSWERS AND RATIONALES 2024 REVISED AND GRADED A+. | Exams Nursing | Docsity (27)

LEIK FNP (642 Practice Questions with 100% Verified Answers and Rationales).

LEIK FNP PRACTICE QUESTIONS WITH ANSWERS AND RATIONALES 2024 REVISED AND GRADED A+. | Exams Nursing | Docsity (28)

LEIK FNP (642 Practice Questions with 100% Verified Answers and Rationales).

LEIK FNP PRACTICE QUESTIONS WITH ANSWERS AND RATIONALES 2024 REVISED AND GRADED A+. | Exams Nursing | Docsity (29)

LEIK FNP (642 Practice Questions with 100% Verified Answers and Rationales).

LEIK FNP PRACTICE QUESTIONS WITH ANSWERS AND RATIONALES 2024 REVISED AND GRADED A+. | Exams Nursing | Docsity (30)

LEIK FNP (642 Practice Questions with 100% Verified Answers and Rationales).

LEIK FNP PRACTICE QUESTIONS WITH ANSWERS AND RATIONALES 2024 REVISED AND GRADED A+. | Exams Nursing | Docsity (31)

CCRN PRACTICE QUESTIONS WITH 100% PRECISE ANSWERS AND RATIONALES GRADED A+ 2024 UPDATED( A

LEIK FNP PRACTICE QUESTIONS WITH ANSWERS AND RATIONALES 2024 REVISED AND GRADED A+. | Exams Nursing | Docsity (32)

LEIK FNP PRACTICE QUESTIONS WITH ANSWERS AND RATIONALES 2024 REVISED AND GRADED A+. | Exams Nursing | Docsity (33)

Leik Exam Edge FNP Review Questions with Answers and Rationales 2024.

LEIK FNP PRACTICE QUESTIONS WITH ANSWERS AND RATIONALES 2024 REVISED AND GRADED A+. | Exams Nursing | Docsity (34)

Leik Exam Edge FNP Review Questions with Answers and Rationales 2024.

LEIK FNP PRACTICE QUESTIONS WITH ANSWERS AND RATIONALES 2024 REVISED AND GRADED A+. | Exams Nursing | Docsity (35)

Leik Exam Edge FNP Review Questions with Answers and Rationales 2024.

LEIK FNP PRACTICE QUESTIONS WITH ANSWERS AND RATIONALES 2024 REVISED AND GRADED A+. | Exams Nursing | Docsity (36)

Leik Exam Edge FNP Review Questions with Answers and Rationales 2024.

LEIK FNP PRACTICE QUESTIONS WITH ANSWERS AND RATIONALES 2024 REVISED AND GRADED A+. | Exams Nursing | Docsity (37)

Leik Exam Edge FNP Review Questions with Answers and Rationales 2024.

LEIK FNP PRACTICE QUESTIONS WITH ANSWERS AND RATIONALES 2024 REVISED AND GRADED A+. | Exams Nursing | Docsity (38)

Leik Exam Edge FNP Review Questions with Answers and Rationales 2024.

LEIK FNP PRACTICE QUESTIONS WITH ANSWERS AND RATIONALES 2024 REVISED AND GRADED A+. | Exams Nursing | Docsity (39)

Leik Exam Edge FNP Review Questions with Answers and Rationales 2024.

LEIK FNP PRACTICE QUESTIONS WITH ANSWERS AND RATIONALES 2024 REVISED AND GRADED A+. | Exams Nursing | Docsity (40)

NCLEX RN Practice Test (ALL PASSING LEVEL Questions with Answers and Rationales) 2024 EX

LEIK FNP PRACTICE QUESTIONS WITH ANSWERS AND RATIONALES 2024 REVISED AND GRADED A+. | Exams Nursing | Docsity (41)

CNOR ONLINE PRACTICE EXAM 2024 | ALL QUESTIONS AND CORRECT ANSWERS WITH RATIONALES

LEIK FNP PRACTICE QUESTIONS WITH ANSWERS AND RATIONALES 2024 REVISED AND GRADED A+. | Exams Nursing | Docsity (42)

ILTS 305 Elementary Education Content Exam Questions ,Answers And Rationales 2024 Revised

Partial preview of the text

Download LEIK FNP PRACTICE QUESTIONS WITH ANSWERS AND RATIONALES 2024 REVISED AND GRADED A+. and more Exams Nursing in PDF only on Docsity! LEIK FNP PRACTICE QUESTIONS WITH ANSWERS AND RATIONALES 2024 REVISED AND GRADED A+. A 72-year-old woman has been on hydrochlorothiazide 12.5 mg for many years to control her Stage II hypertension. Her blood pressure (BP) at this visit is 168/96. She is currently complaining of pain on her right hip and on both knees. She has increased her dose of ibuprofen (Motrin) from 400 mg 3 times day (TID) to 800 mg TID. She is still in pain and would like something stronger. Which of the following statements is the best explanation of the effects of ibuprofen (Motrin) on her disease? A) It increases the chances of adverse effects to her health B) It inhibits the effect of renal prostaglandins and blunts the effectiveness of the diuretic C) It prolongs the therapeutic effects of hydrochlorothiazide and other diuretics D) None of the statements are true - Solution B) It inhibits the effect of renal prostaglandins and blunts the effectiveness of the diuretic. NSAIDs and ASA inhibit the vasodilatory effects of prostaglandins, which predisposes the kidney to ischemia. NSAIDs and diuretics can cause acute prerenal failure by decreasing renal blood flow. 2. All of the following are infections that affect mostly the labia and vagin* except: A) Bacterialvaginosis B) Candidiasis C) Trichom*oniasis D) Chlamydia trachomatis - Solution D) Chlamydia trachomatis Infections that commonly affect the labia and vagin* include bacterial vaginosis, candidiasis, and trichom*oniasis. Chlamydia trachomatis commonly affects the cervix, endometrial lining , fallopian tubes, and pelvic cavity. The nurse practitioner would test the obturator and iliopsoas muscle to evaluate for: A) Cholecystitis B) Acute appendicitis C) Inguinal hernia D) Gastriculcer - Solution B) Acute appendicitis. Signs and symptoms of an acute abdomen include invol- untary guarding, rebound tenderness, boardlike abdomen, and a positive obtu- rator and psoas sign. A positive obturator sign occurs when pain is elicted by internal rotation of the right hip from 90 degrees hip/knee flexion. The psoas sign is positive when pain occurs with passive extension of the thigh while the patient is lying on his/her side with knees extended, or when pain occurs with active flexion of his/her thigh at the hip. Treatment for mild preeclampsia includes all of the following except: A) Bed rest except for bathroom privileges B) Close monitoring of weight and blood pressure C) Close follow-up of urinary protein, serum creatinine, and platelet count D) A prescription of methyldopa (Aldomet) to control blood pressure - Solution D) A prescription of methyldopa (Aldomet) to control blood pressure Recommended care for women diagnosed with preeclampsia includes bed rest with bath- room privileges, weight and BP monitoring, and closely following urine protein and serum protein, creatinine, and platelet counts. Oral medications are not used as first-line treatment. All of the following services are covered under Medicare Part A except: A) Inpatienthospitalizations B) Medicines administered to a patient while hospitalized C) Nursing home care D) Surgeons - Solution C) Nursing home care Medicare A coverage includes inpatient hospitalization and skilled care given in a certified skilled nursing facility. Most nursing home care is D) Diuretics - Solution C) Beta-blockers Beta-blockers should be avoided in patients with a history of emphysema. Studies have shown evidence of a reduction in forced expiratory volume in 1 second (FEV1), increased airway hyperresponsiveness, and inhibition of bronchodilator response to beta agonists in patients receiving non-selective beta- blockers and high doses of cardioselective beta-blockers. A 30-year-old chef complains of pruritic hives over her chest and arms but denies difficulty swallowing or breathing. She reports a family history of allergic rhinitis and asthma. Which of the following interventions is most appropriate? A) Perform a complete and thorough history B) Prescribe an oral antihistamine such as diphenhydramine 25 mg PO QID C) Give an injection of epinephrine 1:1000 intramuscularly stat D) Call 911 - Solution A) Perform a complete and thorough history Prior to prescribing medications, a complete and thorough history must be performed to determine possible causes of hives. The patient denied difficulty with swallowing and breathing, so there was no medical emergency to require calling 911. Which of the following findings is most likely in young primigravidas with pregnancy-induced hypertension? A) Abdominal cramping and constipation B) Edema of the face and the upper extremities C) Shortness of breath D) Dysuria and frequency - Solution B) Edema of the face and the upper extremities Common signs and symptoms of pregnancy-induced hypertension include edema of the face and the upper extremities, weight gain, blurred vision, elevated BP, proteinuria, and headaches. Which of the following symptoms is associated with B12 deficiency anemia? A) Spoon-shaped nails and pica B) An abnormal neurological exam C) A vegan diet D) Tingling and numbness of both feet - Solution D) Tingling and numbness of both feet Vitamin B12 deficiency anemia can cause nerve cell damage if not treated. Symptoms of B12 deficiency anemia may include tingling or numbness in fingers and toes, difficulty walking, mood changes or depression, memory loss, disorientation, and dementia. A second triple screen on a 35-year-old primigravida reveals abnormally low lev- els of the alpha fetoprotein and estriol and high levels of human chorionic gonad- otropin. Which of the following interventions is the best choice for this patient? A) Order an ultrasound B) Order a computed tomography (CT) scan of the abdomen C) Order a 24-hour urine for protein clearance D) Assess for a history of illicit drug or alcohol use - Solution A) Order an ultrasound Abnormally low levels of alpha fetoprotein and estriol and high levels of human chorionic gonadotropin are abnormal during pregnancy. An ultrasound should be ordered to further evaluate the fetus for characteristics of Down syndrome and/or fetal demise. All of the following are true statements about diverticula except: A) Diverticula are located in the colon B) A low-fiber diet is associated with the condition C) Most diverticula in the colon are infected with gram-negative bacteria D) Supplementingwithfibersuchaspsyllium(Metamucil)isrecommended - Solution C) Most diverticula in the colon are infected with gram-negative bacte- ria Diverticuli in the colon can be infected with both gram-negative and gram- positive bacteria. Patients who are diagnosed with gonorrhea should also be treated for which of the following infections? A) Chancroid B) Chlamydia trachomatis C) Herpes genitalis D) PID (pelvic inflammatory disease) - Solution B) Chlamydia trachomatis When diagnosed with gonorrhea, the patient should also be treated for Chlamydia trachomatis. Kyphosis is a late sign of: A) Old age B) Osteopenia C) Osteoporosis D) Osteoarthritis - Solution C) Osteoporosis Kyphosis is a curvature of the spine that causes a rounding of the back, which leads to a slouching posture. Severe thinning of the bones (osteoporosis) contributes to this curvature in the spine. Symptoms that may occur with severe cases of kyphosis include difficulty breathing, fatigue, and back pain. A 35-year-old primigravida who is at 28 weeks of gestation is expecting twins. What would you would expect her alpha fetoprotein (AFP) values to be? A) Normal B) Higher than normal C) Lower than normal D) None of the above - Solution B) Higher than normal Alpha fetoprotein is produced in the fetal and maternal liver. Higher levels of alpha fetoprotein are commonly seen in multiple gestations due to the growing fetuses and enlargement of the livers. Which of the following antihypertensive medications has beneficial effects for an elderly White female with osteoporosis? C) A large amount of swelling on the knee D) An injury of the meniscus - Solution B) Instability of the knee The drawer sign is performed on the knee or ankle to assess for knee instability. The affected knee will have more laxity when compared to the unaffected knee. A multigravida who is at 28 weeks of gestation has a fundal height of 29 cm. Which of the following is the best recommendation for this patient? A) Advise the mother that her pregnancy is progressing well B) Order an ultrasound of the uterus C) Refer her to an obstetrician for an amniocentesis D) Recommend bed rest with bathroom privileges - Solution A) Advise the mother that her pregnancy is progressing well From 20-35 weeks' gestation, fundal height should equal weeks gestation. If greater than 2 cm within the gestational week of pregnancy, further testing should be performed to evaluate fetal growth A multigravida who is at 34 weeks of gestation wants to know at what level her uterine fundus should be. The best answer is to advise the mother that her fun- dus is: A) Midway between the umbilicus and the lower ribs B) At the level of the umbilicus C) From 33 to 35 cm D) From 32 to 34 cm - Solution C) From 33 to 35 cm After 20 weeks gestation, fundal height in centimeters should measure approximately the same as the number of weeks of gestation Which of the following laboratory tests is used in primary care to evaluate renal function? A) Electrolyte panel B) Creatinine C) Alkaline phosphatase D) Blood urea nitrogen (BUN) to creatinine ratio - Solution B) Creatinine Serum creatinine is measured to evaluate renal function. Creatinine is the end product of creatine metabolism. Creatinine clearance is not affected by fluid or dietary intake of meat. All of the following are false statements regarding acute gastritis except: A) Chronic intake of nonsteroidal anti-inflammatory drugs (NSAIDs) can cause the disorder B) Chronic lack of dietary fiber is the main cause of the disorder C) The screening test for the disorder is the barium swallow test D) The gold standard to evaluate the disorder is a colonoscopy - Solution C) The screening test for the disorder is the barium swallow test The gold standard for diagnosing gastric disease is biopsy of gastric and/or duodenal tissue by upper endoscopy. Chronic use of NSAIDs disrupts the production of prostaglandins, which decreases blood supply and then breaks down the protective layer of the mucosa, which leads to gastric disease. Signs and symptoms of depression include all of the following except: A) Anhedonia and changes in appetite B) Decreased energy and irritability C) Apathy and low self-esteem D) Apraxia and fatigue - Solution D) Apraxia and fatigue Apraxia is a disorder of the nervous system in which the brain is affected and the patient is unable to move the arms/legs when asked to do so. Common signs of depression include anhedonia (loss of interest in activities that the patient finds pleasurable), unintentional weight loss or gain, fatigue, change in appetite, insomnia or hypersomnia, feelings of guilt and worthlessness, and recurrent thoughts of suicide. Which of the following is an accurate description of eliciting for Murphy's sign? A) Upon deep inspiration by the patient, palpate firmly in the right upper quad- rant of the abdomen below the costovertebral angle B) Bend the patient's hips and knees at 90 degrees, then passively rotate hip exter- nally, then internally C) Ask the patient to squat, then place the stethoscope on the apical area D) Press into the abdomen deeply, then release it suddenly - Solution A) Upon deep inspiration by the patient, palpate firmly in the right upper quad- rant of the abdomen below the costovertebral angle Murphy's sign is positive if pain occurs when performing RUQ palpation while the patient takes a deep inspiration, causing the patient to abruptly stop the deep breath due to pain. As the liver is palpated, it will descend and push up against the inflamed gallbladder and cause sharp pain (cholecystitis). A 28-year-old multipara who is at 32 weeks of gestation presents to your office complaining of a sudden onset of small amounts of bright red vagin*l bleeding. She has had several episodes and appears anxious. On exam, her uterus is soft to palpation. Which of the following is most likely? A) Placenta abruptio B) Placenta previa C) Acute cervicitis D) Molar pregnancy (hydatidiform mole) - Solution B) Placenta previa Placenta previa occurs when abnormal implantation of the placenta occurs. A common symptom of placenta previa is painless, bright red bleeding. Epidemiologic studies show that Hashimoto's disease occurs most commonly in: A) Middle-aged to older women B) Smokers C) Obese individuals D) Older men - Solution A) Middle-aged to older women Hashimoto's disease commonly occurs in middle-aged to older women. A 48-year-old woman is told by a physician that she is starting menopause. All of the following are possible findings except: A) Hot flashes A) Chronic scabies infection B) Lichen sclerosus C) Chronic candidal vaginitis D) A physiologic variant found in some older women - Solution B) Lichen sclerosus Lichen sclerosus is a disease of the skin, in which white spots appear on the skin and change over time. It is most commonly seen in the genital and rectal areas, but can appear in other areas. The spots are usually shiny and smooth and can eventually spread into patches. The skin appears thin and crinkled. Then the skin tears easily, and bright red or purple bruises are common. Sometimes, the skin becomes scarred. If the disease is a mild case, there may be no symptoms The heart sound S2 is caused by: A) Closure of the atrioventricular valves B) Closure of the semilunar valves C) Opening of the atrioventricular valves D) Opening of the semilunar valves - Solution B) Closure of the semilunar valves The heart sound S2 is caused by closure of the semilunar valves. A pelvic exam on a woman who is 12 weeks pregnant would reveal that her uterus is located at which of the following areas? A) Between the umbilicus and the suprapubic bone B) Just rising above the suprapubic bone C) Between the suprapubic bone and the xiphoid process D) Between the umbilicus and the xiphoid proces - Solution B) Just rising above the suprapubic bone At 12 weeks gestation, the uterus measures approximately the size of a grapefruit, which would be felt just above the suprapubic bone on bimanual exam. All of the following are covered under Medicare Part B except: A) Persons age 65 years or older B) Durable medical equipment C) Mammograms annually starting at age 50 D) Anesthesiologist's services - Solution D) Anesthesiologist's services Medicare Part B covers: 1) outpatient physician visits, labs, x-rays; 2) durable medical equipment; 3) mammograms/colonoscopy after age 50 years annually; 4) rehabilitation. Anesthesiologist's services are covered by Medicare Part A. All of the following patients are at higher risk for suicide except: A) A 66-year-old White male whose wife of 40 years recently died B) A high school student with a history of bipolar disorder C) A depressed 45-year-old female with family history of suicide D) A 17-year-old teen who has only 1 close friend in school - Solution D) A 17-year-old teen who has only one close friend in school Risk factors for suicide include: 1) elderly white males (especially after the death of a spouse); 2) past history of suicide; 3) family history of suicide; 4) plans for use of a lethal weapon such as a gun or knife; 5) female gender has a higher attempt rate, but males have a higher success rate; 6) personal history of bipolar disorder or depression. A 70-year-old male patient complains of a bright red spot in his left eye for 2 days. He denies eye pain, visual changes, or headaches. He has a new onset of cough from a recent viral upper respiratory infection. The only medicine he is on is Bayer aspirin 1 tablet a day. Which of the following is most likely? A) Corneal abrasion B) Acute bacterial conjunctivitis C) Acute uveitis D) Subconjunctival hemorrhage - Solution D) Subconjunctival hemorrhage Subconjunctival hemorrhage is a benign disorder that occurs from an increase in intraocular pressure that may be caused by coughing, vomiting, forceful exerrtion in labor during childbirth, straining while having a bowel movement, weight lifting, or lifting a heavy object. Which of the following is appropriate follow-up for this 70-year-old patient? A) Referral to an optometrist B) Referral to an ophthalmologist C) Advise the patient that it is a benign condition and will resolve spontaneously D) Prescribe an ophthalmic antibiotic solution - Solution C) Advise the patient that it is a benign condition and will resolve spontaneously A subconjunctival hemorrhage is a benign disorder that resolves without any treatment Jason, an 8-year-old with type 1 diabetes, is being seen for a 3-day history of fre- quency and nocturia. He denies flank pain and is afebrile. The urinalysis result is negative for blood and nitrites but is positive for a large amount of leukocytes and ketones. He has a trace amount of protein. Which of the following is the best test to order initially? A) Urine for culture and sensitivity B) 24-hour urine for protein and creatinine clearance C) 24-hour urine for microalbumin D) An intravenous pyelogram - Solution A) Urine for culture and sensitivity An 8-year-old male patient with the diagnosis of diabetes has a high risk of urinary tract infections. A large amount of leukocytes in the urinalysis is abnormal and he has been having symptoms of frequency and nocturia for the past 3 days. The urine culture would be ordered because he has a high risk of infection. The urine culture and sensitivity (C&S) is the best evaluation for diagnosing a urinary tract infection. All of the following are not recommended for the outpatient treatment of mild preeclampsia except: A) Severe sodium restriction B) Restrict fluid intake to less than 1 liter per 24 hours C) Aldomet (methyldopa) 250 mg PO (orally) BID D) Bed rest on the left side with bathroom privileges - Solution D) Bed rest on the left side with bathroom privileges Outpatient treatment for mild pre-eclampsia includes bed rest on the left lateral side with bathroom privileges. A 53-year-old crossing guard complains of twisting his right knee while working that morning. The knee is swollen and tender to palpation. The nurse practitioner diagnoses a Grade II sprain. The initial treatment plan includes which of the following? A) Application of cold packs the first 24 hours followed by applications of low heat at bedtime B) Elevation of the affected limb and intermittent applications of cold packs for the next 48 hours C) Rechecking the knee in 24 hours and isometric exercises D) The application of an Ace bandage to the affected kne - Solution B) Elevation of the affected limb and intermittent applications of cold packs for the next 48 hours Elevation of the injured knee above the heart will reduce the amout of swelling that can occur. Use of ice packs immediately after the injury is most effective and will reduce swelling in the tissue. Ice the affected area for 15 minutes at a time intermittently to prevent frostbite and further damage to tissue. Allowing 30-45 minutes between icings of the limb is recommended. Erythromycin 200 mg with sulfisoxazole 600 mg suspension (Pediazole) is contraindicated in which of the following conditions? A) G6PD deficiency anemia B) Lead poisoning C) Beta thalassemia minor D) B12 deficiency anemia - Solution A) G6PD deficiency anemia Glucose-6-phosphate dehydrogenase (G6PD) defi- ciency is a hereditary condition that occurs when the red blood cells break down, causing hemolysis, due to a missing or lack of a sufficient enzyme that is needed to help the red blood cells work efficiently. Certain foods and medications may trigger this reaction and hemolysis will occur. Some of these medications include antimalarial drugs, aspirin, nitrofurantoin, NSAIDs, quinidine, quinine, and sulfa medications. The mother of an 11-year-old boy with sickle cell anemia calls on the phone complaining that her son woke up because of a painful penile erection that will not go away. The nurse practitioner's most appropriate intervention is: A) Insert a Foley catheter and measure the child's intake and output for the next 24 hours B) Insert a Foley catheter to obtain a specimen for a urinalysis and urine for C&S (culture and sensitivity) C) Recommend an increase in the child's fluid intake D) Recommend immediate referral to the ER - Solution D) Recommend immediate referral to the ED Priapism (painful penile erection not related to sexual activity) is a true urologic emergency that may lead to permanent erectile dysfunction and penile necrosis if not treated appropriately. It can be associated with a number of medical conditions (sickle cell anemia, leukemia, or spinal cord injury) and/or some pharmacologic agents A positive Coombs test on an Rh-negative pregnant woman means: A) The mother has autoantibodies against Rh-positive red blood cells (RBCs) B) The fetus has autoantibodies against maternal RBCs C) The mother does not have Rh factor against fetal RBCs D) The fetus does not have RBC autoantibodies - Solution A) The mother has autoantibodies against Rh-positive red blood cells (RBCs) The mother's autoantibodies can attack the fetus's Rh-positive red blood cells and cause destruction of these cells, which can cause severe anemia and com- plications in the fetus. Today this is preventable with the administration of anti- RhD immunoglobulin (Rho(D) immune globulin) to an Rh-negative mother at 28 weeks gestation and after birth if the newborn is Rh positive. Folic acid supplementation is recommended for women who are planning pregnancy in order to: A) Prevent renal agenesis B) Prevent anencephaly C) Prevent kidney defects D) Prevent heart defects - Solution B) Prevent anencephaly Folic acid supplementation during pregnancy has been shown to decrease the risk of neural tube defects of the fetus All of the following are possible causes for secondary hypertension except: A) Acute pyelonephritis B) Pheochromocytoma C) Renovascular stenosis D) Coarctation of the aorta - Solution A) Acute pyelonephritis Pheochromocytoma is an adrenal gland tumor in which there is increased production of the hormones adrenaline and noradrena- line, which may cause long-term elevation in blood pressure. Renovascular steno- sis is a narrowing of one or both arteries leading to the kidneys. It can cause severe hypertension and irreversible kidney damage. Coarctation of the aorta is a con- genital heart defect of the aorta; it is a narrowing of the aorta that causes the heart to work harder to get blood to flow through the narrow aortic passageway to other organs, which in turn causes an increase in blood pressure. Fitz-Hugh-Curtis syndrome is associated with which following infection? A) Syphilis B) Chlamydia trachomatis C) Herpes genitalis D) Lymphogranuloma venereum - Solution B) Chlamydia trachomatis Fitz-Hugh-Curtis is a complication of having pelvic inflammatory disease that was caused by a vagin*l infection, such as gonorrhea or Chlamydia trachomatis. This causes inflammation and infection in the pelvic cavity. Left untreated, this infection can cause adhesions that stretch from the peritoneum to the liver. A nursing home resident reports to his physician that his previous roommate was recently started on tuberculosis treatment. A Mantoux test and chest x-ray are ordered for the patient. What is the minimum size of induration considered positive for this patient? Medicare Part B covers: 1) outpatient physician visits, labs, x-rays; 2) durable medical equipment; 3) mammograms/ colonoscopy after age 50 years annually; and 4) rehabilitation. Which of the following is used to screen for color blindness in a 7-year-old boy? A) Snellen chart B) Ishihara chart C) Cover/uncover test D) Red reflex - Solution B) Ishihara chart The Ishihara chart is used for screening for color blindness. A 67-year-old retired clerk presents with complaints of shortness of breath and weight gain over a 2-week period. A nonproductive cough accompanies her symptoms. The lung exam is positive for fine crackles in the lower lobes with no wheezing. The exam is positive for egophony. Which of the following conditions is most likely? A) Acute exacerbation of asthma B) Left-heart failure C) Right-heart failure D) Chronic obstructive pulmonary disease - Solution B) Left-heart failure Signs and symptoms of left-heart failure include tachypnea, labored breathing, and rales or crackles in the lower bases of the lungs, which can develop into pulmonary edema. Which of the following drugs is most likely to relieve the patient's symptoms? A) Atenolol (Tenormin) B) Trimethoprim/sulfamethoxazole (Bactrim DS) C) Furosemide (Lasix) D) Hydrocodone/guaifenesin syrup (Hycotuss) - Solution C) Furosemide (Lasix) Lasix would be used to help remove the extra fluid load. A patient has recently been diagnosed with migraine headache with aura. She wants advice on what she should eliminate from her diet to reduce her risk of headaches. The best response is to advise this patient to avoid: A) Foods with aspartame (Equal) because they can trigger migraines B) Foods with monosodium glutamate because they promote diuresis C) Foods with garlic because they promote hypertension D) Foods with nitrates because they do not induce flushing - Solution A) Foods with aspartame (Equal) because they can trigger migraines Dietary triggers for migraine headaches include foods that contain aspartame, tyramine, nitrites, MSG, and/or red wine. These foods are thought to cause a change in the blood vessels and increased blood flow to the brain. A patient with a history of mitral valve prolapse (MVP) is requesting prophylaxis before her dental surgery. Which of the following would you prescribe this patient? A) Amoxicillin a half hour before and 2 hours after the procedure B) Amoxicillin 1 hour before the procedure C) Amoxicillin 1 hour before and 3 hours after the procedure D) Prophylaxis is not recommended for this patient - Solution D) Prophylaxis is not recommended for this patient The American Heart Association does not recommend routine antibiotic prophylaxis prior to dental procedures except for patients who are at high risk for bacterial endocarditis (a prosthetic cardiac valve, previous history of bacterial endocarditis, congenital heart defects, etc.). Koplik's spots are associated with: A) Poxvirus infections B) Measles C) Kawasaki's disease D) Reye's syndrome - Solution B) Measles Signs and symptoms of the measles include fever over 101°F, coryza, cough, conjunctivitis, rash, and Koplik's spots on buccal mucosa. Stella, a new mother, complains to you that she has been feeling irritable and jittery almost daily for the past few months. She complains of frequent palpitations and more frequent bowel movements along with weight loss. Her BP is 160/70, her pulse is 110, and she is afebrile. All of the following conditions should be considered in the differential diagnosis for this patient except: A) Mitral regurgitation B) Graves' disease C) Generalized anxiety disorder D) Illicit drug use - Solution A) Mitral regurgitation Signs and symptoms of mitral regurgitation do not include frequent bowel movements with weight loss. An elderly patient with a productive cough and fever is diagnosed with pneumonia. All of the following organisms are capable of causing community-acquired pneumonia except: A) Haemophilus influenzae B) Mycoplasma pneumoniae C) Treponema pallidum D) Streptococcus pneumoniae - Solution C) Treponema pallidum Treponema pallidum is a gram-negative spirochete bacterium that causes syphilis Human chorionic gonadotropin (hCG) is produced by the: A) Placenta B) Hypothalamus C) Anterior pituitary D) Ovaries - Solution A) Placenta HCG is produced by the placenta. The majority of serum alpha fetoprotein is produced by the: A) Palpation of the fetus and auscultation of the fetal heart tones by the nurse practitioner B) Palpation of the fetus and a positive quantitative serum pregnancy test C) Fetal heart tones and a positive quantitative serum pregnancy test D) Fetal heart tones and feeling of movement of the baby by the mother - Solution A) Palpation of the fetus and auscultation of the fetal heart tones by the nurse practitioner Presumptive signs of pregnancy are symptoms experienced by the woman, such as amenorrhea, breast tenderness, nausea/vomiting, fatigue, and increased urinary frequency. Probable signs of pregnancy are signs detected by the examiner, such as an enlarged uterus. Positive signs of pregnancy are direct evidence of pregnancy such as audible fetal heart tones or cardiac activity on ultrasound. Chadwick's sign is characterized by: A) Softening of the cervix B) Blue coloration of the cervix and vagin* C) Softening of the uterine isthmus D) Nausea and vomiting during the first trimester of pregnancy - Solution B) Blue coloration of the cervix and vagin* Chadwick's sign is defined as a bluish discoloration of the cervix and vagin*. These changes are caused by the increased vascularity and congestion in the pelvic area during pregnancy. A college student has recently been informed that he has an HPV (human papilloma virus) infection on the shaft of his penis. Which of the following may reveal subclinical lesions on the penile skin? A) Perform a KOH (potassium hydroxide) exam B) Scrape off some of the affected skin and send it for a culture and sensitivity C) Apply acetic acid to the penile shaft and look for acetowhite changes D) Order a serum herpes virus titer - Solution C)Apply acetic acid to the penile shaft and look for acetowhite changes Lesions of HPV infection will turn white with application of acetic acid. Carol M. is a 40-year-old bank teller who has recently been diagnosed with obsessive-compulsive disorder by her therapist. Her symptoms would include: A) Ritualistic behaviors that the patient feels compelled to repeat B) Attempts to ignore or suppress the repetitive behaviors, which increase anxiety C) Frequent intrusive and repetitive thoughts and impulses D) All of the above - Solution D) All of the above Signs and symptoms of obsessive-compulsive disorder include ritualistic behaviors that are repeated, increased anxiety with attempting to ignore repetitive behaviors, and frequent intrusive and repetitive thoughts and impulses. Which of the following medications is indicated for the treatment ofobsessive-compulsive disorder? A) Paroxetine (Paxil CR) B) Haldoperidol (Haldol) C) Lorazepam (Xanax) D) Imipramine (Elavil - Solution A) Paroxetine (Paxil CR) Antidepressants are the most common medications used for OCD. Those antidepressants that are approved for OCD by the Food and Drug Administration (FDA) include clomipramine (Anafranil), fluvox- amine (Luvox), fluoxetine (Prozac), paroxetine (Paxil, Pexeva), and sertraline (Zoloft). You would advise an 18-year-old female student who has been given a booster dose of MMR at the college health clinic that: A) She might have a low-grade fever during the first 24 to 48 hours B) She should not get pregnant within the next 4 weeks C) Her arm will be very sore on the injection site for 24 to 48 hours D) Her arm will have some induration on the injection site in 24to48hours - Solution B) She should not get pregnant within the next 4 weeks MMR should not be administered to women known to be pregnant. In addition, women should be counseled to avoid becoming pregnant for 28 days following vaccination. Jean, a 68-year-old female, is suspected of having Alzheimer's disease. Which of the following is the best initial method for assessing the condition? A) Computed tomography scan of the brain B) Mini Mental Status Exam C) Obtain the history from the patient, friends, and family members D) EEG (electroencephalography) - Solution B) Mini Mental Status Exam Begin with administering the MMSE for a baseline assessment, followed by taking a history from the patient and others. A 55-year-old woman who has type 2 diabetes is concerned about her kidneys. She has a history of 3 urinary tract infections within the past 8 months. She denies dysuria and frequency at this visit. Which of the following is the best initial course to follow? A) Recheck the patient's urine and order a urine for culture and sensitivity B) Order an IVP (intravenous pyelogram) C) Advise the patient to follow up with a urologist D) Evaluate the patient for a possible kidney infection - Solution A) Recheck the patient's urine and order a urine for culture and sensitivity A urinary tract infection is defined as the presence of 100,000 organisms per mL of urine in asymptomatic patients or greater than 100 organisms per mL or urine with pyuria (more than 7 WBCs/mL) in a symptomatic patient A nurse practitioner is giving dietary counseling to a male alcoholic who has recently been diagnosed with folic acid deficiency anemia. Which of the following foods should the nurse practitioner recommend to this patient? A) Tomatoes, oranges, and bananas B) Cheese, yogurt, and milk C) Lettuce, beef, and dairy products D) Spinach, liver, and whole wheat bread - Solution D) Spinach, liver, and whole wheat bread B) Records are subpoenaed C) Reports are sent to the Public Health Department D) Records are released to insurance companies - Solution A) Medical information is given to a spouse Patient confidentiality is breached when medical information is given to a spouse or any other individual without consent of the patient. A 25-year-old woman's last menstrual period was 6 weeks ago. She is complaining of nausea with vomiting in the morning and fatigue. Her breasts feel bloated. The nurse practitioner suspects that she is pregnant. Her symptoms would be considered: A) Positive signs of pregnancy B) Probable signs of pregnancy C) Presumptive signs of pregnancy D) Possible signs of pregnancy - Solution C) Presumptive signs of pregnancy Presumptive signs of pregnancy are symptoms experienced by the woman, such as amenorrhea, breast tenderness, nausea/ vomiting, fatigue, and increased urinary frequency. A 55-year-old male patient describes to you an episode of chest tightness in his substernal area that radiated to his back while he was jogging. It was relieved immediately when he stopped. Which of the following conditions does this best describe? A) Angina pectoris B) Acute myocardial infarction C) Gastroesophageal reflux disease D) Acute costochondritis - Solution A) Angina pectoris Symptoms of angina pectoris include a pressing, squeezing, or crushing pain, usually in the chest or back, arms, neck, or ear lobes. Weakness, fatigue, and shortness of breath and pain radiating in the arms, shoulders, jaw, neck, and/or back may also occur. Angina pectoris is most commonly brought on by physical exertion, but can also be triggered by stress and other stressors to the body. Which of the following would you recommend to this 55-year-old patient? A) Start an exercise program by starting with walking instead of jogging B) Consult with a cardiologist for further evaluation C) Consult with a gastroenterologist to rule out acute cholecystitis D) Take ibuprofen (Advil) 600 mg for pain every 4 to 6 hours PRN - Solution B) Consult with a cardiologist for further evaluation Physical assessment and examination by a cardiologist are advised to rule out cardiac problems and deter- mine the safety of physical exercise. Carol, a 73-year-old patient, complains of episodic vertigo, slight confusion, and weakness that last nearly an hour each time. Movement does not worsen the vertigo. She "rests" and her symptoms subside, but she is puzzled because the weakness "jumps from side to side," sometimes on the right and sometimes on the left of her body. Her symptoms suggest: A) Benign paroxysmal positional vertigo B) Ménière's disease C) TIA (transient ischemic attack) D) CVA (cerebrovascular accident) - Solution C) TIA (transient ischemic attack) Transient ischemic attack (TIA) is caused by vascular occlusion. Symptoms of a TIA usually last less than 1 hour; however, the symptoms may also be permanent. Common signs/symptoms include neurologic deficits, vertigo, confusion, weakness, hemiparesis, temporary monocular blindness, ataxia, and diplopia. All of the following measures have been found to help lower the risk of osteoporosis except: A) Drinking organic juice B) Eating low-fat dairy foods C) Performing weight-bearing exercises D) Vitamin D supplementation - Solution A) Drinking organic juice Vitamin D levels must be sufficient for the body to absorb calcium. Eating foods high in vitamin D and calcium along with calcium and vitamin D supplements are advised to protect the bones and prevent bone loss. Performing weight-bearing exercises daily also increases bone strength. A 28-year-old male nurse of Hispanic descent reports a history of a cold that resolved 2 weeks ago except for a dry cough and pain over his right cheek that worsens when he bends down. The patient denies fever. He tells the employee health nurse practitioner that he is very allergic to both cephalexin (Keflex) and erythromycin. The vital signs show that the patient's temperature is 99.2 degrees Fahrenheit, a pulse of 72 beats per minute, and a respiratory rate of 12 breaths per minute. Which of the following conditions is most likely? A) Acute sinusitis B) Acute bronchitis C) Fever secondary to the previous viral URI (upper respiratory infection) D) Munchausen's syndrome - Solution A) Acute sinusitis Acute sinusitis symptoms include cough, facial pain, and low-grade fever. A 38-year-old multigravida who is at 32 weeks of gestation calls the family nurse practitioner complaining of bright red vagin*l bleeding. There is no watery dis- charge. She complains that her uterus feels hard and is very painful. Which of the following conditions is most likely? A) Placenta previa B) Placenta abruptio C) A molar pregnancy D) An ectopic pregnancy - Solution B) Placenta abruptio Abruptio placenta symptoms are bright red vagin*l bleeding, board-like uterus on palpation, and pain. However, there can be concealed hemorrhage and the patient may not have vagin*l bleeding. Placenta previa is painless bleeding. Ectopic and molar pregnancy would not progress to 32 weeks gestation The Jarisch-Herxheimer reaction is best described as: A) An immune-mediated reaction precipitated by the destruction of a large number of spirochetes due to an antibiotic injection B) Severe chills and elevated blood pressure C) Acute pyelonephritis D) Renal stenosis - Solution D) Renal stenosis Renal stenosis is a narrowing of the renal artery. No blood would be noted on exam. Evidence of blood in the urine can be seen with kidney stones, bladder cancer, and acute pyelonephritis. During a routine physical exam of an elderly woman, a triangular thickening of the bulbar conjunctiva on the temporal side is noted to be encroaching on the cornea. She denies any eye pain or visual changes. Which of the following is most likely? A) Corneal arcus B) Pterygium C) Pinguecula D) Chalazion - Solution B) Pterygium Pterygium is a triangular growth on the white part of the eye that also extends onto the cornea. Corneal arcus is a ring around the edge of the cornea. Chalazion is a stye in the eye that may cause pain and swelling. Pinguecula is a benign growth on the conjuctiva caused by the degeneration of its collagen fibers. Thick, yellow fibers may be seen. Mr. Jones, who has been on pravastatin (Pravachol) 20 mg at bedtime for the past few months, complains of lately feeling extremely fatigued. The patient also noticed that his urine has been a darker color during the past 2 weeks. Which of the following is the best treatment plan to follow? A) Discontinue his pravastatin and order a liver function profile B) Continue the pravastatin but on half the dose C) Schedule him for a complete physical exam D) Schedule him for a liver function profile - Solution A) Discontinue his pravastatin and order a liver function profile Side effects of statin drugs include myalgia, fatigue, and elevated liver enzymes. Routine labs should include liver enzymes to assess the effects on the liver. Jane, a young primigravida, reports to you that she is starting to feel the baby's movements in her uterus. This is considered to be which of the following? A) Presumptive sign B) Probable sign C) Positive sign D) Possible sign - Solution B) Probable sign Probable signs of pregnancy are symptoms of pregnancy that are felt by the woman. A 14-year-old teen is worried that she has not started to menstruate like most of her friends. During the gynecological examination, the nurse practitioner tells the mother, who is in the room with the patient, that her daughter is starting Tanner Stage II. What are the physical exam findings during this stage? A) Breast buds and some straight pubic hair B) Fully developed breasts and curly pubic hair C) Breast tissue with the areola on a separate mound with curly pubic hair D) No breast tissue and no pubic hair - Solution A) Breast buds and some straight pubic hair Tanner Stage II in females includes breast buds and few straight, fine pubic hairs. The Phalen test is used to evaluate: A) Inflammation of the median nerve B) Rheumatoid arthritis C) Degenerative joint changes D) Chronic tenosynovitis - Solution A) Inflammation of the median nerve The Phalen test is used to identify inflammation of the median nerve seen with carpal tunnel syndrome. Phalen's test is performed by examining the wrist with full flexion of the wrist for 60 seconds. Positive findings are reproduction of the symptoms such as numbness and tin- gling of the thumb, index finger, and middle finger areas. Physiologic anemia of pregnancy is due to: A) An increase in the cardiac output at the end of the second trimester B) A physiologic decrease in the production of RBCs in pregnant women C) An increase of up to 50% of the plasma volume in pregnant women D) An increase in the need for dietary iron in pregnancy - Solution C)An increase of up to 50% of the plasma volume in pregnant women Physiologic anemia of pregnancy is caused by the increased volume of plasma during pregnancy when compared to the production of RBCs. All of the following are implicated in causing chronic cough except: A) Chronic bronchitis B) Allergic rhinitis C) Acute viral upper respiratory infection D) Gastroesophageal reflux disease - Solution C) Acute viral upper respiratory infection Chronic cough can be caused by chronic bronchitis, allergic rhinitis, and GERD. Which of the following is a true statement regarding the effect of aspirin on platelet function? A) The effect on platelets is reversible B) The effect on platelets is reversible and lasts only 1 week C) It has a minimal effect on platelet function D) The effect on platelet function is irreversible and lasts 15 to 20 days - Solution D) The effect on platelet function is irreversible and lasts 15 to 20 days The use of aspirin affects the platelet function, is irreversible, and can last up to 15-20 days All of the following agents are used to control the inflammatory changes seen in the lungs of asthmatics except: A) Albuterol inhaler (Proventil) B) Triamcinolone (Azmacort) A) Cytomegalovirus (CMV) B) Epstein-Barr virus (EBV) C) Human papilloma virus (HPV) D) Coxsackie virus - Solution B) Epstein-Barr virus (EBV) A test for an acute infection of the Epstein-Barr virus will show abnormal forms of lymphocytes. A faun tail nevus is a sign of which of the following? A) Down syndrome B) Infantile scoliosis C) Congenital heart disease D) Spina bifida - Solution D) Spina bifida A faun tail nevus is an abnormal tuft of hair in the lumbosacral area, which can be a sign of spina bifida. Jenny, a 21-year-old, complains to you of a 1-week episode of dysuria, frequency, and a strong odor to her urine. This is her second episode of the year. What is the most appropriate follow-up for this patient? A) Order a urinalysis and urine for culture and sensitivity (C&S) and treat the patient with antibiotics B) Order a urine C&S and hold treatment until you get the results from the lab C) Treat the patient with a 7-day course of antibiotics and order a urine for culture and sensitivity (urine C&S) now and after she completes her antibiotics D) Treat the patient with a stronger drug such as ofloxacin (Floxin) for 10 days - Solution A) Order a urinalysis and urine for culture and sensitivity (C&S) and treat the patient with antibiotics The best treatment for this patient is to order the urinalysis and urine C&S to identify the organism causing the infection. Treatment may begin while waiting for the culture results. However, occasionally treatment may have to be changed after the culture/sensitivity results return, due to resistance to the antibiotic used. Café-au-lait spots look like tan-to-light brown stains that have irregular borders. They can be located anywhere on the body. Which of the following is a correct statement? A) They are associated with neurofibromatosis or von Recklinghausen's disease B) They may be considered as precancerous after a biopsy C) They are more common in darker-skinned children D) They are associated with Wilson's disease - Solution A) They are associated with neurofibromatosis or von Recklinghausen's disease Café-au-lait spots are caused by an increase in melanin content, often with the presence of giant melanosomes. They have irregular borders and vary in color from light to dark brown. Neurofibromatosis causes tumors to grow in the nervous system, and these tumors commonly cause skin changes that are seen as café-au-lait spots. During the eye exam of a 50-year-old hypertensive patient who is complaining of an onset of a severe headache, you find that the borders of the disc margins on both eyes are blurred. What is the name of this clinical finding? A) Normal optic disc B) Optic neuropathy C) Papilledema D) Hypertensive retinopathy - Solution C) Papilledema Papilledema is defined as having optic disc swelling that is caused by increased intracranial pressure. It is commonly seen in both eyes and can develop over a time period of hours or weeks. Headache is a common complaint. A high school teacher complains of a dry cough for the past 6 weeks. It worsens when he is supine. He has episodes of heartburn, which he self- treats with an over- the-counter (OTC) antacid. He chews mints for his "bad breath." Which of the fol- lowing is a possible cause for this patient's cough? A) Asthma B) Gastroesophageal reflux C) Pneumonia D) Chronic postnasal drip - Solution B) Gastroesophageal reflux GERD is a condition in which food comes up from the stomach/esophagus due to a weak sphincter. It usually worsens with lying down and can cause a cough and esophageal irritation if not treated. The red reflex is elicited by shining a light in the eyes of the infant at an angle with the light about 15 in. away. The nurse practitioner is screening for? A) Cataracts B) Strabismus C) Blindness D) The blinking response - Solution A) Cataracts Screening for cataracts is performed by shining a light into the eyes at an angle, approximately 15 inches away. If the red reflex is not elicited, then cataracts are present. A 44-year-old patient with Down syndrome starts to develop impaired memory and difficulty with his usual daily life routines. He is having problems functioning at the job that he has done for the past 10 years. The physical exam and routine labs are all negative. The vital signs are normal. His appetite is normal. The most likely diagnosis is: A) Tic douloureux B) A stroke C) Alzheimer's disease D) Delirium - Solution C) Alzheimer's disease Delirium is an acute decline in mental status and is temporary. Common causes are fever, shock, drugs, alcohol, and dehydration. Alzheimer's disease is a permanent change to the brain that causes short-term memory loss, agnosia, apraxia, and aphasia. In this case, the patient's physical exam is normal; however, he is having memory loss and difficulty working and carrying out his normal tasks heart rate is regular and no murmurs are auscultated. Which of the following is correct? A) This is an abnormal finding and should be evaluated further by a cardiologist B) A stress test should be ordered C) This is a normal finding in some young athletes D) An echocardiogram should be ordered - Solution C) This is a normal finding in some young athletes It is common to hear a split of the S2 over the pulmonic area of the heart with inspiration as long as it disappears with expiration, with no other abnormal symptoms. This is caused by split- ting of the aortic and pulmonic components. Mrs. J. L. is a 55-year-old female with a body mass index (BMI) of 24 and a history of asthma. She has hypertension that has been under control with hydrochlorothiazide 12.5 mg PO daily. Her total cholesterol is 230 g/dl. How many risk factors for coronary heart disease (CAD) does she have? A) 1 risk factor B) 2 risk factors C) 3 risk factors D) 4 risk factors - Solution C) 3 risk factors Risk factors for coronary heart disease include hypertension, hypercholesterolemia, and women 55 years of age and older A common side effect of metformin (Glucophage) therapy is: A) Weight gain B) Lactic acidosis C) Hypoglycemic episodes D) Gastrointestinal problems - Solution D) Gastrointestinal problems Common side effects of metformin include diarrhea/gastrointestinal problems. While doing a cardiac exam on a 45-year-old male, you note an irregular rhythm with a pulse rate of 110 beats per minute. The patient is alert and is not in distress. What is the most likely diagnosis? A) Atrial fibrillation B) Ventricular fibrillation C) Cardiac arrhythmia D) First-degree right bundle branch block - Solution A) Atrial fibrillation Atrial fibrillation is chaotic electrical activity of the heart, caused by several ectopic foci in the atria without any signs of distress. The following are patients who are at high risk for complications due to urinary tract infections. Who does not belong in this category? A) A 38-year-old diabetic patient with a HbA1C of 7.5% B) A woman with a history of rheumatoid arthritis who is currently being treated with a regimen of methotrexate and low-dose steroids C) A 21-year-old woman who is under treatment for 2 sexually transmitted infections D) Pregnant women - Solution C) A 21-year-old woman who is under treatment for 2 sexually transmitted infections Risk factors for complications due to urinary tract infection include pregnancy, diabetes, and steroid therapy. A 68-year-old woman with hypertension and diabetes is seen by the nurse practitioner for a dry cough that worsens at night when she lies in bed. She has shortness of breath, which worsens when she exerts herself. The patient's pulse rate is 90/min and regular. The patient has gained 6 lbs over the past 2 months. She is on a nitro- glycerine patch and furosemide daily. The best explanation for her symptoms is: A) Kidney failure B) Congestive heart failure C) ACE inhibitor-induced coughing D) Thyroid disease - Solution B) Congestive heart failure Signs and symptoms of congestive heart failure (CHF) include dyspnea on exertion, edema, fatigue, hemoptysis, cough, orthopnea, hypertension, and nocturnal dyspnea. A nurse practitioner is doing a funduscopic exam on a 35-year-old female during a routine physical exam. He notices that she has sharp disc margins and a yellowish- orange color in the macular area. The ratio of veins to arteries is 3:2. What is the next most appropriate action? A) Advise the patient that she had a normal exam B) Advise the patient that she had an abnormal exam C) Refer the patient to the emergency room D) Refer the patient to an ophthalmologist - Solution A) Advise the patient that she had a normal exam A normal funduscopic exam would include sharp disc margins, with the macula appearing yellow-orange in color. The ratio comparing the artery to vein width is 2:3. During a sports physical, you note that the vision of an 18-year-old male athlete is 20/30 in both eyes. Which of the following statements is true? A) The patient can see at 20 ft what a person with normal vision can see at 30 ft B) The patient can see at 30 ft what a person with normal vision can see at 20 ft C) The patient cannot engage in contact sports D) The patient needs to be referred to an ophthalmologist - Solution A) The patient can see at 20 ft what a person with normal vision can see at 30 ft When vision results are 20/30 in both eyes, this means that the patient can see at 20 ft what a person with normal vision can see at 30 ft. Carol, a 30-year-old type 2 diabetic, is on regular insulin and lente insulin in the morning and in the evening. She denies changes in her diet or any illness, but recently started attending aerobic classes in the afternoon. Because of her work- outs, her blood sugars have dipped below 50 mg/ dL very early in the morning. D) Educate her on how to patch the infant's eye every 4 hours - Solution B) Advise the mother that this is a normal finding in infants up to 2 months of age Infants' eyes commonly cross over at times, and this is a normal finding up to 2 months of age. All of the following are factors important in determining the peak expiratory flow volume except: A) Weight B) Height C) Age D) Gender - Solution A) Weight Peak expiratory flow volume is determined by using height, gender, and age. You are reviewing a Pap smear report on a 25-year-old female. Which of the following cells should be on a Pap smear to be classified as a satisfactory specimen? A) Clue cells and endometrial cells B) vagin*l cells and cervical cells C) Squamous epithelial cells and endocervical cells D) Leukocytes and RBCs - Solution C) Squamous epithelial cells and endocervical cells Squamous epithelial cells and endocervical cells must be obtained when performing a Pap smear to be considered satisfactory to evaluate the cells from the endocervix. When assessing a patient suspected of having vertigo, which description provided by the patient is most consistent with the diagnosis? A) A sensation of imbalance while walking B) A sensation of spinning or rotating C) A sensation of "passing out" D) A sensation of lightheadedness when changing positions from reclining to standing - Solution B) A sensation of spinning or rotating Vertigo is defined as having a sensation of spinning or rotating A 56-year-old man complains of episodes of lancinating pain that shoots up to his right cheek when he eats or drinks. He has stopped drinking cold drinks because of the pain. Which of the following is most likely? A) Trigeminal neuralgia B) Cluster headache C) Acute sinusitis D) Sinus headache - Solution A) Trigeminal neuralgia Pain shooting up the right cheek with food or drink is seen with trigeminal neuralgia. What does a KOH (potassium hydroxide) prep help the nurse practitioner diagnose? A) Herpes zoster infections B) Yeast infections C) Herpes simplex infections D) Viral infections - Solution B) Yeast infections The KOH prep test is performed by placing a sample of dis- charge on a glass slide, with one drop of potassium hydroxide and a coverslip on top to evaluate for yeast infections. Budding spores and pseudohyphae will be seen with Candida, with a pH of 3.5-4.5. All of the following are considered selective serotonin reuptake inhibitors (SSRIs) except: A) Imipramine (Elavil) B) Fluoxetine (Prozac) C) Sertraline (Zoloft) D) Paroxetine (Paxil CR) - Solution A) Imipramine (Elavil) Prozac, Zoloft, and Paxil CR are selective serotonin reuptake inhibitors. Elavil is a tricyclic antidepressant. All of the following tests require the patient's voice to perform correctly except: A) Egophony B) Tactile fremitus C) Whispered pectoriloquy D) Auscultation - Solution D) Auscultation Egophony, tactile fremitus, and whispered pectoriloquy require the patient to speak. Auscultation is listening to an organ with the use of a stethoscope. While checking for the red reflex on a 3-year-old boy during a well child visit, a white reflection is seen on the child's left pupil. Which of the following conditions should be ruled out? A) Unilateral strabismus B) Unilateral cataracts C) Retinoblastoma of the left eye D) Color blindness of the left eye - Solution C) Retinoblastoma of the left eye Retinoblastoma is a rare type of cancer in which a cancerous tumor of the retina is present. This is diagnosed by noting a pupil that appears white or has white spots on it. One or both eyes may be affected. It is often seen in photographs, where there will be a white glow in the eye instead of the usual "red eye" noted in pictures from the flash. All of the following infections are reportable diseases except: A) Lyme disease B) Gonorrhea C) Nongonococcal urethritis D) Syphilis - Solution C) Nongonococcal urethritis The public health department requires all agencies to report Lyme disease, gonorrhea, and syphilis. Statistics of these reportable diseases are kept in each state. A 14-month-old child should developmentally be able to say "mama" and "dada," know his own name, and know at least 2-4 words. A 2-year-old is able to understand simple commands. You are reviewing the bilirubin level on a 3-day-old full-term neonate. You note that it is 10 mg/dL. The infant has a slight yellow color to his skin, mucous membranes, and sclera. The infant is feeding well, is not irritable, and has 8 to 10 wet diapers per day. Which of the following is a true statement? A) Keep on monitoring the infant's bilirubin level until it returns back to normal in about 1 week B) Recommend that the infant be treated with phototherapy 10 minutes a day until the bilirubin level is back down to a normal range C) Refer the infant to a neonatologist as soon as possible D) Refer the infant to the neonatal intensive care unit - Solution A) Keep on monitoring the infant's bilirubin level until it returns back to normal in about 1 week Bilirubin is excreted through the urine and feces. Increased fluids and wetting 8-10 diapers a day is sufficient fluid intake/excretion to help bring down the bilirubin level. Levels should continue to be monitored and should improve in approximately 1 week. All of the following statements are correct regarding the Td vaccine except: A) Fever occurs in up to 80% of the patients B) A possible side effect is induration on the injection site C) The Td is given every 10 years D) The DPT and DT should not be given beyond the seventh birthday - Solution A) Fever occurs in up to 80% of the patients Side effects of the Td vaccine include induration at the injection site. Td is given in adults every 10 years. The DPT and DT should not be given beyond 7 years of age. Fever may occur, but studies do not support 80% of patients having fever. Which of the following is recommended by JNC 7 as first-line treatment for hypertension in patients with microalbuminuria? A) Angiotensin-converting enzyme (ACE) inhibitors B) Diuretics C) Calcium channel blockers D) Beta-blockers - Solution A) Angiotensin-converting enzyme (ACE) inhibitors First line of treatment for hypertension in patients with microalbuminuria is ACE inhibitors. ACE inhibitors are renal protective. A woman is being evaluated by the nurse practitioner for complaints of dyspareunia. A microscopy slide reveals a large number of atrophic squamous epithelial cells. The vagin*l pH is 4.0. There are very few leukocytes and no RBCs are seen on the wet smear. Which of the following is most likely? A) Atrophic vaginitis B) Bacterial vaginosis C) Trichom*oniasis D) This is a normal finding - Solution A) Atrophic vaginitis Symptoms of atrophic vaginitis include painful intercourse, atrophic squamous epithelial cells, and a decrease in pH. vagin*l atrophy is caused by lack of or imbalance of estrogen. Normal pH of the vagin* is 4.0-5.0 (acidic). A test called the visual fields by confrontation is used to evaluate for: A) Peripheral vision B) Central distance vision C) Narrow-angle glaucoma D) Accommodation - Solution A) Peripheral vision The visual fields of confrontation test is used to evaluate peripheral vision. The Snellen chart is used to measure central distance vision. A tonometer is used to assess for glaucoma. The ophthalmoscope is used to assess for cataracts. The following skin findings are considered macules except: A) A freckle B) Petechiae C) Acne D) A flat, 0.5-cm brown birthmark - Solution C) Acne A macule is a flat, non-raised lesion on the skin. Acne lesions are pap- ules because they consist of raised, erythemic lesions on the skin. A freckle, petechiae, and a flat birthmark are all considered macules. Clara is a 20-year-old college student who reports to the student health clinic with a laceration in her left hand. She tells the nurse practitioner that she cut her hand while working in her garden. Her last Td booster was 5.5 years ago. Which of the following is correct? A) Administer a booster dose of the Td vaccine B) Administer the Td vaccine and the Td immunoglobulin (HyperTet) C) Administer Td immunoglobulin (HyperTet) only D) She does not need any Td immunoglobulin (Hypertet) or a Td booster - Solution A) Administer a booster dose of the Td vaccine Td booster vaccine is recommended for recent injuries if it has been more than 5 years since the last Td. The apex of the heart is located at: A) Second ICS (intercostal space) to the right of the sternal border B) Second ICS to the left of the sternal border C) The left lower sternal border D) The left side of the sternum at the fifth ICS by the midclavicular line - Solution D) The left side of the sternum at the fifth ICS by the midclavicular line The apex of the heart is located at the left side of the sternum at the fifth ICS by the midclavicular line. Elderly patients with type 2 diabetes should have a dilated eye exam done annually by an ophthalmologist. It is also recommended to see a podiatrist once or twice a year. Preventive care also includes receiving a flu shot annually, receiving a pneumovax vaccine if over 60 years of age, and taking a baby aspirin of 81 mg each day. A 72-year-old female complains to you of a crusty and non-healing small ulcer on her upper lip that she has had for several years. Which of the following would you recommend? A) Triamcinolone acetonide (Kenalog) cream BID for 2 weeks B) Triple antibiotic ointment BID x 2 weeks C) Hydrocortisone 1% cream BID for 2 weeks D) The patient needs to be evaluated by a dermatologist - Solution D) The patient needs to be evaluated by a dermatologist Non healing ulcers of the skin are a risk for skin cancer and should be evaluated by a dermatologist for treatment. All of the following statements about phototherapy are correct except: A) Light from the blue to white spectrum is used B) It is not always necessary to use a shield for the infant's eyes C) Unconjugated bilirubin in the skin is converted to a water-soluble nontoxic substance that is excreted in the bile D) The infant's eyes should be shielded - Solution B) It is not always necessary to use a shield for the infant's eyes When using phototherapy, the eyes should always be protected by using a shield or goggles to prevent damage to the eyes. Heberden's nodes are commonly found in which of the following diseases? A) Rheumatoid arthritis B) Degenerative joint disease C) Psoriatic arthritis D) Septic arthritis - Solution B) Degenerative joint disease Heberden's nodes are bony nodules on the distal interphalangeal joints, commonly seen in degenerative joint disease. The red blood cells in pernicious anemia will show: A) Microcytic and hypochromic cells B) Microcytic and normochromic cells C) Macrocytic and normochromic cells D) Macrocytic and hypochromic cells - Solution C) Macrocytic and normochromic cells The blood cells in pernicious anemia will display macrocytic, normochromic cells. RBCs in iron-deficiency anemia are microcytic and hypochromic. You notice a medium-pitched harsh systolic murmur during an episodic exam. It is best heard at the right upper border of the sternum. What is most likely? A) Mitral stenosis B) Aortic stenosis C) Pulmonic stenosis D) Tricuspid regurgitation - Solution B) Aortic stenosis Aortic stenosis is best heard at the right upper border of the sternum, radiating to the neck with a medium-pitched systolic murmur. Mitral stenosis is heard at the apex of the heart and sounds like a low-pitched diastolic rumbling murmur. It is suggested that this murmur is heard best with the bell side of the stethoscope. A small abscess on a hair follicle on the eyelid is called: A) Hordeolum B) Pterygium C) Pinguecula D) Ptosis - Solution A) Hordeolum Hordeolum is a painful, acute bacterial infection of the hair follicle on the eyelid. Pinguecula is a thickening of the bulbar conjunctiva, located on the inner and outer margins of the cornea. Pterygium is a thickening of the conjunctiva located on the nasal or temporal cornea. Pinguecula and pterygium are both caused by the UV light of long-term sun exposure. Sunglasses with UV protection are recommended to prevent damage to the conjunctiva. Ptosis is the drooping of the upper eyelid. Which of the following is indicated for the prophylactic treatment of migraine headache? A) Ibuprofen (Motrin) B) Naproxen sodium (Anaprox) C) Propranolol (Inderal) D) Sumatriptan (Imitrex) - Solution C)Propranolol (Inderal) Medications used to prophylactically prevent migraine headaches include beta-blockers (propranolol) and tricyclic antidepressants (ami- triptyline). Motrin, Anaprox, and Imitrex are all medications used to treat migraine headache. A 40-year-old male complains to the nurse practitioner of severe stabbing pains behind his left eye for the past 2 days. It is accompanied by some nasal congestion and rhinorrhea, which is clear in color. The patient denies pharyngitis and fever. Which of the following conditions is most likely? A) Migraine headache with aura B) Cluster headache C) Tic douloureux D) Cranial neuralgia - Solution B) Cluster headache Signs/symptoms of cluster headaches include severe stabbing pain behind the eyes, with nasal congestion and rhinorrhea. Migraine head- aches with aura include visual changes, such as blind spots or flashing lights that appear before the onset of the headache. Trigeminal neuralgia (tic douloureux) is a unilateral headache from compression or inflammation of the trigeminal nerve (cranial nerve 5). Ken has type 2 diabetes mellitus and a "sensitive stomach." Which medication is least likely to cause him gastrointestinal distress? A) Naproxen sodium (Anaprox) B) Aspirin (Bayer's aspirin) C) Erythromycin (E-mycin) A) Cardiovascular system B) Neurological system C) Gastrointestinal system D) Renal system - Solution D) Renal system The 2 main adverse drug reactions associated with NSAIDs relate to gastrointestinal (GI) effects and renal effects of the agents. The main adverse drug reactions associated with use of NSAIDs relate to direct and indirect irritation of the GI tract. Ulceration risk increases with therapy duration and with higher doses. NSAIDs can induce 2 different forms of acute kidney injury: hemodynamically mediated, and acute interstitial nephritis, which is often accompanied by nephrotic syndrome. Ted, who is 15 years old, has just moved into the community and is staying in a foster home temporarily. There is no record of his immunizations. His foster mother wants him to be checked before he enters the local high school. Which of the fol- lowing does this patient need? A) Meningococcal B) MMR C) Tdap D) All of the above - Solution D) All of the above The measles, mumps, and rubella (MMR) is recommended as one of the "catch-up" immunizations for this patient's age group. The tetanus immunization that is recommended as a "catch-up" for that age group is the Tdap instead of just the Td. Which cranial nerve (CN) is being evaluated when patients are instructed to shrug their shoulders? A) CN IX B) CN X C) CN XI D) CN XII - Solution C) CN XI Cranial nerves IX, X, XI, and XII are: glossopharyngeal, vagal, spinal accessory, and hypoglossal, respectively. A lab technician has a 10.5-mm area of redness and induration in his left forearm after getting a Mantoux test 72 hours ago. The last test, which was done 12 months ago, was negative. He denies cough, night sweats, and weight loss. What is the next best intervention? A) Obtain a sputum culture and a chest x-ray B) Obtain a chest x-ray C) Obtain a sputum for C&S and an acid fast stain D) Obtain a CBC and chest x-ray - Solution B) Obtain a chest x-ray Mantoux results are negative if results show less than 10 mm induration on the forearm for low-risk clients. If results are greater or equal to 10 mm, then a chest x-ray should be ordered for further screening. Swim therapy for a 13-year-old with cerebral palsy is an example of: A) Primary prevention B) Secondary prevention C) Tertiary prevention D) Health prevention - Solution C) Tertiary prevention Tertiary prevention is any type of rehabilitation for a particular condition. Examples include physical rehab (swimming), cardiac rehab, and/or speech therapy. Which of the following individuals is most likely to be at higher risk for osteoporosis? A) 70-year-old female of African ancestry who walks daily for exercise B) 42-year-old obese woman from Cuba who has been taking prednisone 10 mg daily for the last 12 years to control her severe asthma C) 55-year-old Caucasian female who is an aerobics instructor D) 4-year-old Asian female who has been on high-dose steroids for 1 week - Solution B) 42-year-old obese woman from Cuba who has been taking prednisone 10 mg daily for the last 12 years to control her severe asthma Risk factors for osteoporosis include postmenopause, early menopause, use of chronic steroids, smoking, excessive use of alcohol, sedentary lifestyle, insufficient intake of calcium and Vitamin D in the diet, and being an Asian or Caucasian female. What is the primary carbohydrate found in breast milk and commercial infant formulas? A) Fructose B) Lactose C) Glucose D) Sucrose - Solution B) Lactose Lactose is the primary carbohydrate found in breast milk and formula Which of the following patients is least likely to become an alcoholic? A) A patient whose father has a history of alcoholism B) A patient whose wife complains that he drinks too much C) A patient who drinks one cup of wine nightly with dinner D) A patient who feels he drinks all the time - Solution C) A patient who drinks one cup of wine nightly with dinner Excessive use or exposure to alcohol puts that patient at risk for becoming an alcoholic. A patient who drinks one glass of wine at dinner has a lower risk of becoming an alcoholic than someone who has been exposed to alcohol while growing up, one who drinks all the time, or one whose family believes he is drinking excessively. The following conditions are absolute contraindications for the use of oral contraceptives except: A) Hepatomas B) History of emboli that resolved with heparin therapy 15 years ago C) A family history of migraines with aura D) A history of gallbladder disease during pregnancy - Solution C) A family history of migraines with aura All of the items are contraindications, but a family history does not substantiate a need to avoid oral contraceptives. The most common cause of cancer deaths in males is: C) An increased risk of urinary tract infections D) Bladder cancer - Solution A) Loss of joint mobility and renal failure Complications of untreated gout are loss of joint mobility and renal failure. High uric acid levels can also lead to kidney stones. A 21-year-old female who is complaining of random palpitations is diagnosed with mitral valve prolapse (MVP). Her echocardiogram (EKG) reveals redundant and thickened leaflets. You note a Grade III/VI systolic murmur with an ejection click during physical examination. You would recommend: A) Endocarditis prophylaxis for most dental and urologic procedures B) Endocarditis prophylaxis is not necessary C) Lifetime anticoagulation therapy with warfarin sodium D) Endocarditis prophylaxis for dental procedures only - Solution B) Endocarditis prophylaxis is not necessary Prophylaxis treatment for endocarditis is no longer recommended for MVP. You note bony nodules located at the proximal interphalangeal joints on both the hands of your 65-year-old female patient. Which of the following is most likely? A) Bouchard's node B) Heberden's node C) Osteoarthritic nodules D) Tophi deposits - Solution A) Bouchard's node Bony nodules at the proximal interphalangeal joints of the hands are called Bouchard's nodes. Heberden's nodes are nodules on the distal interphalangeal joints. Tophi deposits are seen with gout, where high levels of uric acid in the blood occur and cause nodules in the joint that can eventually destroy the bone. Osteoarthritic nodules develop in the joints of the hands. Which chronic illness disproportionately affects the Hispanic population? A) Diabetes mellitus B) Hypertension C) Alcohol abuse D) Skin cancer - Solution A) Diabetes mellitus Diabetes mellitusis 2-3 times higher in Mexican Americans versus non- Hispanic Americans. A lipid profile done on a newly diagnosed hypertensive patient shows a triglyceride level of 650 mg/dL, total cholesterol 240 mg/dL, LDL 145 mg/dL, and an HDL of 35 mg/dL. What is the best intervention for this patient? A) Educate the patient about lifestyle changes that will help lower cholesterol levels B) Initiate a prescription of pravastatin (Pravachol) C) Recommend that the patient exercise at least every other day and avoid eating fatty or fried foods D) Initiate a prescription of nicotinic acid (Niacin, Niaspan) - Solution D) Initiate a prescription of nicotinic acid (Niacin, Niaspan) Niacin is recommended for treatment of high triglyceride and cholesterol levels. Exercise and healthy eating lifestyle is also recommended, but with the high level of triglycerides at 650 mg/dL and total cholesterol 240 mg/dL, niacin is recommended. Cullen's sign is most commonly associated with which of the following? A) Acute pancreatitis B) Myocardial infarction C) Acute pyelonephritis D) Preeclampsia - Solution A) Acute pancreatitis Cullen's sign is commonly seen in acute pancreatitis. It is a yellowish-blue skin color change around the umbilicus. It is thought to occur due to the pancreatic enzymes that run along the ligament and subcutaneous tis- sues around the umbilicus. The "gold standard" for the diagnosis of active Helicobacter pylori infection of the stomach or duodenum is: A) A Helicobacter pylori titer B) An endoscopy with tissue biopsy C) An upper GI series D) A urea breath test - Solution B) An endoscopy with tissue biopsy The "gold standard" for diagnosing Helicobacter pylori infection is to perform tissue biopsy via endoscopy. Which of the following foods would you advise a new mother to introduce to her 6-month-old infant first? A) Iron-fortified rice cereal B) Plain rice cereal C) Iron-fortified pureed chicken meat D) Plain pureed carrots - Solution A) Iron-fortified rice cereal At 6 months of age, it is recommended that the infant be fed iron-fortified rice cereal. Introducing only one new food at a time is recommended; in case an allergy does develop, you will be able to identify which food caused the allergy. Jim Wheeler is obese (BMI of 33), fatigued, and complaining of excessive thirst and hunger. You suspect type 2 diabetes mellitus. Initial testing to confirm diagnosis can include: A) Fasting plasma glucose level B) Glycosylated hemoglobin level (HbA1 c) C) Glucose tolerance testing D) All of the above - Solution D) All of the above Type 2 diabetes mellitus screening tests include: fasting plasma glucose level (> 126 mg/dL), random plasma glucose level (> 200 mg/dL), and glucose tolerance testing (2 hr blood glucose level > 200 mg/dL) with 75 g glucose load. Normal HbA1c levels are < 6% A bulla is defined as: A) A solid nodule less than 1 cm in size B) A superficial vesicle filled with serous fluid greater than 1 cm in size Genital ulcers may occur with syphilis, genital herpes, and chancroid. Molluscum contagiosum is a viral infection that causes smooth, round tiny papules, approximately 5 mm or less, that have a central umbilication with a white plug present. Lead poisoning can cause which type of anemia? A) A mild macrocytic anemia B) Normocytic anemia C) Microcytic anemia D) A mild hemolytic anemia - Solution C) Microcytic anemia Lead poisoning can cause microcytic anemia. Signs and symptoms of lead poisoning are abdominal pain, constipation, vomiting, and blue-black line on the gums. Lead causes anemia by mimicking healthful minerals such as calcium, iron, and zinc. It is absorbed by the bones, where it interferes with the production of RBCs. This absorption can also interfere with calcium absorption that is needed to keep the bones healthy. Hypovolemic shock would most likely occur with fractures of the: A) Spine B) Pelvis C) Femur D) Humerus - Solution B) Pelvis Hypovolemic shock can occur from a fractured pelvis secondary to internal bleeding from a fractured bone fragment that lacerates an artery or vein. The pelvis can accommodate a large amount of fluid. Podagra is associated with which of the following? A) Rheumatoid arthritis B) Gout C) Osteoarthritis D) Septic arthritis - Solution B) Gout Podagra is pain in the joint of the great toe due to the accumulation of uric acid and salts in the joint. While assessing for a cardiac murmur, the first time that a thrill can be palpated is at: A) Grade II B) Grade III C) Grade IV D) Grade V - Solution C) Grade IV A thrill is palpated at grade IV. A medium-pitched harsh mid-systolic murmur is best heard at the right second ICS of the chest. It radiates into the neck. Which of the following is the correct diagnosis? A) Aortic stenosis B) Pulmonic stenosis C) Aortic regurgitation D) Mitral stenosis - Solution A) Aortic stenosis The murmur associated with aortic stenosis can be auscul- tated as harsh and high pitched in the right second intercostal space; it typically radiates to the carotid arteries and apex. Which type of hepatitis virus infection is more likely to result in chronic hepatitis and increased risk of developing hepatocellular carcinoma? A) Hepatitis Avirus B) Hepatitis B virus C) Hepatitis C virus D) Both hepatitis B and hepatitis C - Solution D) Both hepatitis B and hepatitis C Of the primary hepatitis viruses, only B and C are associated with hepatocellular cancer. What is the caloric content of infant formula and breast milk? A) 10 kcal/30 mL B) 15 kcal/30 mL C) 20 kcal/30 mL D) 25 kcal/30 mL - Solution C) 20 kcal/30 mL The caloric content of infant formula and breast milk is 20k cal/30 mL. A 19-year-old female has recently been diagnosed with acute hepatitis B. She is sexually active and is monogamous. She reports using condoms inconsistently. What would you recommend for her male sexual partner who was also tested for hepatitis with the following results: HBsAg (−), anti- HBs (−), anti-HCV (−), anti-HAV (+)? A) A hepatitis B vaccination B) Hepatitis B immunoglobulin C) Hepatitis B vaccination and hepatitis B immunoglobulin D) No vaccination is needed at this time - Solution C) Hepatitis B vaccination and hepatitis B immunoglobulin Hepatitis B vaccination is given for long-term prophylaxis treatment to prevent hepatitis B infection. Hepatitis B immunoglobulin is given for prevention of hepatitis B infection when the person has been directly exposed to the hepatitis B infection. The immnoglobulin is not a vaccine and does not protect against long-term prophylaxis. All of the following conditions are associated with an increased risk for normocytic anemia except: A) Rheumatoid arthritis B) Lupus C) Chronic autoimmune disorders D) Pregnancy - Solution D) Pregnancy During pregnancy, women may experience microcytic, hypochromic anemia due to the dilutional effect of the increased blood volume during the pregnancy. You can determine a pulse deficit by counting the: A) Apical and radial pulses at the same time, then finding the difference between the two B) Apical pulse first, then the radial pulse, and subtracting to find the difference between the two C) Apical pulse and the femoral pulse at the same time and finding the difference between the two C) Aspartate aminotransferase and alanine aminotransferase D) Serum sodium, potassium, and magnesium - Solution B) Serum creatinine and potassium levels Native Americans have a much higher rate of kidney disease and renal failure when compared to other races. Native Americans have a 1 in 3 incidence of hypertension. Hypertension is the second leading cause of kidney failure. BUN, creatinine, estimated GFR, and urinalysis are performed to assess the function of the kidneys. You suspect an enterobiasis infection in a 6-year-old girl. Which of the following tests would you recommend? A) Stool culture and sensitivity B) Stool for ova and parasites C) The scotch tape test D) A Hemoccult test - Solution C) The scotch tape test Enterobiasis infection (pinworms) is caused by small worms that infect the intestines. Symptoms include itching around the anus, which is usually worse at night. The scotch tape test is done by applying the scotch tape on the anal area in the morning; the worms commonly come out at night and will stick to the tape, which is used for diagnosis. What is the most common cause of left ventricular hypertrophy in the United States? A) Chronic atrial fibrillation B) Chronic hypertension C) Mitral valve prolapse D) Pulmonary hypertension - Solution B) Chronic hypertension The most common cause of LVH is chronic hypertension. Asthmatics may have all of the following symptoms during an exacerbation except: A) Rapid pulse B) Wheezing C) Chronic coughing D) Tachypnea - Solution C) Chronic coughing Signs and symptoms of an acute exacerbation of asthma include breathlessness, coughing, wheezing, and chest tightness. Agitation, increased respiratory and pulse rate, and decreased lung function are also noted. A "chronic" cough is not a symptom. Laws governing nurse practitioner authority are determined by the: A) Board of Nursing B) State Legislature C) American Nursing Association D) State Medical Association - Solution A) Board of Nursing Each state has laws establishing a Board of Nursing that regulates the role of the nurse practitioner. A 10-year-old male who was recently accepted into his school's soccer team has a history of exercise-induced asthma. The child wants to know when he should take his albuterol inhaler. The nurse practitioner would advise the patient: A) Premedicate himself 20 minutes prior to starting exercise B) Wait until he starts to exercise before using the inhaler C) Premedicate 60 minutes before starting exercise D) Wait until he finishes his exercise before using his inhaler - Solution A) Premedicate himself 20 minutes prior to starting exercise Exercise-induced asthma is best controlled by using the Proventil inhaler (bronchodilator) approximately 20 minutes prior to exercise, to prevent vasospasm of the bronchioles and shortness of breath with exercise. These bronchodilators usually last approximately 4 hours. They also work quickly to open up the bronchioles if an acute attack/shortness of breath occurs Atrophic macular degeneration of the aged (AMD) is the leading cause of blindness in the elderly in the United States Which of the following statements is correct? A) It is a slow or sudden painless loss of central vision B) It is a slow or sudden painless loss of peripheral vision C) It is an occlusion of the central retinal vein causing degeneration of the macular area D) It is commonly caused by diabetic retinopathy - Solution A) It is a slow or sudden painless loss of central vision Macular degeneration causes the loss of central vision. A 12-year-old girl is complaining of a 2-week history of facial pressure that worsens when she bends over. She complains of tooth pain in her upper molars on the right side of her face. On physical exam, her lung and heart sounds are normal. Which of the following is the most likely diagnosis? A) An acute dental abscess B) Chronic sinusitis C) Acute sinusitis D) Severe allergic rhinitis - Solution C) Acute sinusitis Signs/symptoms of acute sinusitis includes headache, facial pain that worsens with bending over, eye/ear pressure and pain, aching in upper jaw/teeth, reduced smell and taste, cough (especially at night due to the nasal drainage), sore throat, bad breath, and fatigue. Which of the following findings is associated with thyroid hypofunction? A) Graves disease B) Eye disorder C) Thyroid storm D) Myxedema - Solution D) Myxedema Myxedema is a rare, and sometimes fatal, disease in which the thyroid is severely underactive and causes life-threatening symptoms. These symptoms include low blood pressure, decreased breathing, decreased body temperature, unresponsiveness, and even coma. Graves' disease, B) Free T4 test C) Antimicrosomal antibody test D) Any of the above - Solution D) Any of the above In addition to conducting a physical examination and taking a thorough history and symptoms into account, 1 or more laboratory tests are used to diagnose Hashimoto's thyroiditis. The 3 most common diagnostic tests that detect this common thyroid disorder are: serum thyroid-stimulating hormone test (TSH), anti-thyroid antibodies tests, and the free T4 hormone test. A first-grader presents to a school nurse practitioner with a few blisters on one arm and on the face. The child keeps scratching the affected areas. Some of the lesions have ruptured with yellow serous fluid that crusts easily. This best describes: A) Acute cellulitis B) Herpes zoster C) Bullous impetigo D) Erysipelas - Solution C) Bullous impetigo Bullous impetigo is a skin infection, commonly seen on the face and hands, with yellow, honey-colored fluid blisters with drainage that turn into scabbed lesions that continue to spread until treated. All of the following statements reflect inadequate breast milk production except: A) Full-term infant is at birth weight by the second week of life B) Less than 6 wet diapers per day or less than 4 stools per day C) Infant is nursing fewer than 8 times per 24-hour period D) Weight loss of > 10% of birth weight - Solution A) Full-term infant is at birth weight by the second week of life The full- term infant should be back to birth weight at 2 weeks of age. The infant should be nursing every 2-4 hours and should wet 6-10 diapers a day (24 h). The best screening test for detecting and monitoring both hyperthyroidism and hypothyroidism is: A) The total T3-4 B) Thyroid-stimulating hormone (TSH) C) Thyroid profile D) Palpation of the thyroid gland - Solution B) Thyroid-stimulating hormone (TSH) The best "screening" test of hypothyroidism and hyperthyroidism is the TSH. If this result is abnormal, then further diagnostic tests should be performed. A patient who recently returned from a vacation in Latin America complains of a severe headache and stiff neck that are accompanied by a high fever for the past 12 hours. While examining the patient, the nurse practitioner flexes both the patient's hips and legs and then tells the patient to straighten them against resistance. The name of this test is: A) Kernig's maneuver B) Brudzinski's maneuver C) Murphy's sign D) Homan's sign - Solution A) Kernig's maneuver Kernig's maneuver is performed by having the patient flex both hips and legs and then straighten the legs against resistance, testing for meningitis. Flexion of hip/knees is a positive sign for meningitis. Brudzinski's maneuver is performed by placing the patient's hands behind his head, and gently tucking chin to chest. Murphy's sign is elicited by having the patient inspire with the tips of the examiner's fingers placed on the right upper quadrant, at the liver border, under the ribs. Pain on inspiration is suggestive of cholecystitis. Homan's sign is flexion of the foot, causing pain in the posterior calf area, suggestive of a DVT. Which of the following groups has been recommended to be screened for thyroid disease? A) Women 50 years or older B) Adolescent females C) Elderly males D) School-age children - Solution A) Women 50 years or older Screening for thyroid disease is recommended for women 50 years of age and older. A 65-year-old Hispanic woman has a history of type 2 diabetes. A routine urinalysis reveals a few epithelial cells and is negative for leukocytes, nitrites, and protein. Which of the following would you recommend next? A) Order a urine for C&S B) Order a 24-hour urine for microalbumin C) Because it is negative, no further tests are necessary D) Recommend a screening IVP (intravenous pyelogram) - Solution B) Order a 24-hour urine for microalbumin Epithelial cells are cells that are present on the inside lining of the organs. Few epithelial cells can be normal; however, with her history of type 2 diabetes, a 24-hour urine for microalbumin should be ordered to assess kidney function. RhoGAM's mechanism of action is: A) The destruction of Rh-positive fetal RBCs that are present in the mother's circulatory system B) The destruction of maternal antibodies against Rh-positive fetal RBCs C) The stimulation of maternal antibodies so that there is a decreased risk of hemolysis D) The destruction of maternal antibodies against fetal RBCs - Solution A) The destruction of Rh-positive fetal RBCs that are present in the mother's cirulatory system RhoGAM is given to mothers with Rh-negative blood when the fetus has Rh-positive blood. RhoGAM protects the mother from developing antibodies by destroying the Rh-positive fetal RBCs in the mother's blood system. the left side of the midclavicular line. Which of the following is the correct diagnosis? A) Aortic regurgitation B) Mitral stenosis C) Mitral regurgitation D) Tricuspid regurgitation - Solution B) Mitral stenosis Mitral stenosis is best heard at the fifth ICS to the left of the midclavicular line. It is ausculated as a low-pitched diastolic murmur, grade II/ VI. Aortic regurgitation is a high-pitched diastolic murmur, heard at the second ICS to the right of the sternum. Mitral regurgitation is a pansystolic murmur that radiates to the axilla, loud and high pitched when ausculated. Which of the following situations is considered emergent? A) A laceration on the lower leg of a patient on aspirin (Bayer) 81mg every other day B) Rapid breathing and tachycardia in a patient with a fever C) An elderly man with abdominal pain whose vital signs appear stable D) A 37-year-old male biker with a concussion due to a fall who appears slightly agitated and does not appear to understand instructions given by the medical assistant checking his vital signs - Solution D) A 37-year-old male biker with a concussion due to a fall who appears slightly agitated and does not appear to understand instructions given by the medical assistant checking his vital signs The biker who had the concussion is the emergent situation due to his agitation and his inability to follow directions, which could mean he has some type of brain trauma. The laceration can be treated with pressure to stop the bleeding until help arrives. The man with abdominal pain has normal vital signs. Tachycardia is common with fever. Which of the following is considered an objective finding in patients who have a case of suppurative otitis media? A) Erythema of the tympanic membrane B) Decreased mobility of the tympanic membrane as measured by tympanogram C) Displacement of the light reflex D) Bulging of the tympanic membrane - Solution B) Decreased mobility of the tympanic membrane as measured by tympanogram Fluid behind the ear drum will decrease the mobility of the TM when measured by a tympanogram. When perforation occurs, discharge will flow through the pars tensa portion of the ear drum. Pulsus paradoxus is more likely to be associated with: A) Sarcoidosis B) Acute bronchitis C) Status asthmaticus D) Bacterial pneumonia - Solution C) Status asthmaticus Pulsus paradoxus is most likely to be seen with status asthmaticus. With inspiration, systolic pressure drops due to the increased pres- sure (positive pressure). Some pulmonary risks of having increased pressure include asthma and emphysema. Cardiac causes for pulsus paradoxus include tamponade, pericarditis, and cardiac effusion. A 17-year-old boy reports feeling something on his left scrotum. On palpation, soft and movable blood vessels that feel like a "bag of worms" are noted underneath the scrotal skin. It is not swollen or reddened. The most likely diagnosis is: A) Chronic orchitis B) Chronic epididymitis C) Testicular torsion D) Varicocele - Solution D) Varicocele Palpation of varicose veins, known as "bag of worms" in the scrotum, is a classic symptom of a varicocele. Chronic epididymitis and chronic orchi- tis are caused by a bacterial infection and commonly have burning, frequency, and pain. Testicular torsion is an emergent condition in which the testicl* becomes twisted, interrupting the blood supply to the testis; to avoid damage, the condition must be corrected within 6 hours. All of the following are true statements regarding elder abuse except: A) Those aged 80 years or older are at the highest risk for abuse B) A delay in medical care is a common finding C) A new onset of an STD in an elderly patient may signal sexual abuse D) Decreased anxiety and depression are common symptoms of abuse in the elderly - Solution D) Decreased anxiety and depression are common symptoms of abuse in the elderly Elder abuse is commonly seen in elderly patients over the age of 80 years. Common signs/symptoms include anxiety and depression. A new onset of an STD may indicate signs of sexual abuse. These patients will commonly delay treatment for acute/chronic conditions. The S1 heart sound is caused by: A) Closure of the atrioventricular valves B) Closure of the semilunar valves C) Opening of the atrioventricular valves D) Opening of the semilunar valves - Solution A) Closure of the atrioventricular valves Closure of the AV valves causes the S1 sound heard when auscultating heart sounds. Patients with Down syndrome are at higher risk for all of the following except: A) Atlantoaxial instability B) Congenital heart disease C) Early onset of Alzheimer's disease D) Melanoma - Solution D) Melanoma Patients with the diagnosis of Down syndrome are at higher risk for atlantoaxial instability, congenital heart defects, and early onset of Alzheimer 's disease. Children with Down syndrome who participate in sport activities must be carefully examined for these conditions prior to participation to prevent injury. Duvall and Miller used developmental theory to describe the family. Which of the following statements is true?

Documents

Summaries

Exercises

Exam

Lecture notes

Thesis

Study notes

Schemes

Document Store

View all

questions

Latest questions

Biology and Chemistry

Psychology and Sociology

Management

Physics

University

United States of America (USA)

United Kingdom

LEIK FNP PRACTICE QUESTIONS WITH ANSWERS AND RATIONALES 2024 REVISED AND GRADED A+. | Exams Nursing | Docsity (43)

Sell documents

Seller's Handbook

About us

Career

Contact us

Partners

How does Docsity work

Koofers

Español

Italiano

English

Srpski

Polski

Русский

Português

Français

Deutsch

United Kingdom

United States of America

India

Terms of Use

Cookie Policy

Cookie setup

Privacy Policy

Sitemap Resources

Sitemap Latest Documents

Sitemap Languages and Countries

Copyright © 2024 Ladybird Srl - Via Leonardo da Vinci 16, 10126, Torino, Italy - VAT 10816460017 - All rights reserved

LEIK FNP PRACTICE QUESTIONS WITH ANSWERS AND RATIONALES 2024 REVISED AND GRADED A+. | Exams Nursing | Docsity (2024)

References

Top Articles
Latest Posts
Article information

Author: Gov. Deandrea McKenzie

Last Updated:

Views: 6299

Rating: 4.6 / 5 (46 voted)

Reviews: 85% of readers found this page helpful

Author information

Name: Gov. Deandrea McKenzie

Birthday: 2001-01-17

Address: Suite 769 2454 Marsha Coves, Debbieton, MS 95002

Phone: +813077629322

Job: Real-Estate Executive

Hobby: Archery, Metal detecting, Kitesurfing, Genealogy, Kitesurfing, Calligraphy, Roller skating

Introduction: My name is Gov. Deandrea McKenzie, I am a spotless, clean, glamorous, sparkling, adventurous, nice, brainy person who loves writing and wants to share my knowledge and understanding with you.